PT81.S2.Q25 - Substantial economic growth must be

accordingcake071accordingcake071 Core Member
edited April 2022 in Logical Reasoning 121 karma

I understand why D is correct, but can someone give an example of what C looks like in an argument? How would you be able to determine whether the evidence given is "stronger" than what the conclusion requires?

Admin Note: https://7sage.com/lsat_explanations/lsat-81-section-2-question-25/

Comments

  • Chris NguyenChris Nguyen Alum Member Administrator Sage 7Sage Tutor
    edited April 2022 4577 karma

    For flaw questions, I like to employ JY's step process in the ACs:

    I ask myself two questions:
    1) Does this describe what's happening in the argument? (If the answer is no, eliminate the AC. If the answer is yes, move on to question 2.)
    2) Is this a flaw? (If the answer is no, eliminate the AC. If the answer is yes, it's the correct answer.)

    With (C), giving more information than the conclusion requires can look something like this:

    Premise: All animals are cute.
    Conclusion: Therefore, all dogs are cute.

    Essentially, in the premises, we're given extra information that we don't need. We only need to conclude that dogs are cute, but the premises gave us way more than that, telling us about all animals, including cats, zebras, and sloths, etc etc. So the premise is "stronger", or essentially gives more information than what the conclusion requires to be true.

    Asking question 1 first, this doesn't describe what's happening in the argument because the premises aren't giving us something that's "stronger" than what's required to make the conclusion true. In actuality, the premises stated aren't even enough to make the conclusion true. I'd eliminate this AC off of this question under timed conditions.

    Even if you ask question 2, however, this AC wouldn't be considered a flaw in the way the LSAT understands flaws. Arguments would be flawed if the stated premises do not make the conclusion true. But giving more information than the conclusion needs will still make the conclusion true. So it's wrong on the basis of both questions.

    Hope this helps!

Sign In or Register to comment.